Solve the following.
2x^2-7x-4/6x^2+7x+2<0
Can someone explain how to solve this.

Answers

Answer 1

Answer:

Which one have same variable gather or decrease up

Step-by-step explanation:

2x^2-4.6x^2=1.4x^2

-7x+7x=0

1.4x^2+2<0

x<0


Related Questions

You order CDs for $14.25 each and the website charges $4.50 for each shipment.
The expression $14.25p + $4.50 represents the cost of p CDs. Find the total cost for
ordering 4 CDs.

Answers

Answer:

$61.50

Step-by-step explanation:

14.25(4) + 4.50

= 57.00 + 4.50

= 61.50

A box contains a yellow ball, an orange ball, a green ball, and a blue ball. Billy randomly selects 4 balls from the box (with replacement). What is the expected value for the number of distinct colored balls Billy will select?

Answers

Answer:

[tex]Expected = 0.09375[/tex]

Step-by-step explanation:

Given

[tex]Balls = 4[/tex]

[tex]n = 4[/tex] --- selection

Required

The expected distinct colored balls

The probability of selecting one of the 4 balls is:

[tex]P = \frac{1}{4}[/tex]

The probability of selecting different balls in each selection is:

[tex]Pr = (\frac{1}{4})^n[/tex]

Substitute 4 for n

[tex]Pr = (\frac{1}{4})^4[/tex]

[tex]Pr = \frac{1}{256}[/tex]

The number of arrangement of the 4 balls is:

[tex]Arrangement = 4![/tex]

So, we have:

[tex]Arrangement = 4*3*2*1[/tex]

[tex]Arrangement = 24[/tex]

The expected number of distinct color is:

[tex]Expected = Arrangement * Pr[/tex]

[tex]Expected = 24 * \frac{1}{256}[/tex]

[tex]Expected = \frac{3}{32}[/tex]

[tex]Expected = 0.09375[/tex]

The 12th term of the arithmetic sequence is 10.5. The 18th term of this sequence is 13.5. Find the common difference and the first term.

Answers

1st way:

let the n'th term be called x, and the value of the term y,

then there is a function y=a*x + b

that will give us all term we want.

this formula is also used for straight lines.

we just need a and b. we already got two data points. we can just plug the known x/y pairs into the formula

10.5 = a*12 + b

13.5 = a*18 + b

now lets manipulate these lines.

multiply the first line by 3 and the second line by 2

31.5 = a*36 + 3b

27 = a*36 + 2b

subtract the second line from the first line

4.5 = b

yey, we now know b, let's plus b into either line from above, I'll go with the first one, looks easier.

10.5 = a*12 + 4.5

6 = a*12

0.5 = a

now y=a*x + b can be filled with a and b

y = 0.5 * x + 4.5

for x=1 (the first term) it's f(1)=5

each step is 0.5, hence the common difference.

2and way:

(13.5-10.5)/(18-12)

= 3 / 6

= 0.5

we got the comment difference by looking at the full difference over 6 steps and divined by these 6 steps.

from step 12 to step one it's 11 steps down

10.5 - 0.5*11

= 10.5 - 5.5

= 5

okay I admit... 2and way to do it might be faster and more intuitive... :D

Find a 2-digit number smaller than 50, the sum of whose digits does not change after being multiplied by a number greater than 1

Answers

The only 2-digit number that is lesser than 50 and the sum of its digits remain unaffected despite being multiplied by a number < 1 would be '18.'

To prove, we will look at some situations:

If we add up the two digits of 18. We get,

[tex]1 + 8 = 9[/tex]

And we multiply 18 by 2 which is greater than 1. We get,

[tex]18[/tex] × [tex]2 = 36[/tex]

The sum remains the same i.e. [tex]3 + 6 = 9[/tex]

Similarly,

If 18 is multiplied to 3(greater than 1), the sum of the two digits comprising the number still remains the same;

[tex]18[/tex] × [tex]3 = 54[/tex]

where (5 + 4 = 9)

Once more,

Even if 18 is multiplied to 4 or 5(greater than 1), the sum of its digits will still be 9.

 [tex]18[/tex] × [tex]4 = 72[/tex]

[tex](7 + 2 = 9)[/tex]

[tex]18[/tex] × [tex]5 = 90[/tex]

[tex](9 + 0 = 9)[/tex]

Thus, 18 is the answer.

Learn more about 'numbers' here: brainly.com/question/1624562

In the PQRS triangle PQ=QR, QR side extended to S Show that PQ+RS=QS. -S Q R

pls explain too

Answers

Answer:

Step-by-step explanation:

from the picture:

QP = QR

and

QR = RS

so

PQ + RS = QS

indentify the explicit function for the sequence in the table ​

Answers

Answer:

Option B

a(n) = 8 + (n-1) • 6

Answered by GAUTHMATH

working alone, aliyah can dig a 10ft by 10 ft hole is 6 hours. one day her friend eugene helped her and it only took 3.75 hours. how long would it take eugene to do it alone? disclaimer: the answer is 11.95 hours, but i keep getting 10. please use the 1/x+1/y=1/z strategy

Answers

Answer:

I think the answer is 10 hours

the 11.95 seems wrong

Step-by-step explanation:

[tex]\frac{1}{6} + \frac{1}{y} = \frac{1}{3.75}[/tex]

multiply by 22.5 y

3.75 y + 22.5 = 6y

y = 10

given m||n, find the value of x

Answers

Answer:

x = 155º

Step-by-step explanation:

y = 25°    {Vertically opposite angles}

x + y = 180    {Co-interior angles are supplementary}

x + 25 = 180

Subtract 25 from both sides

x = 180 - 25

x = 155°

Write down at least five number pairs to solve the equation vw = 50

Answers

Answer:

1, 50

2, 25

5, 10

10, 5

25, 2

Answer:

Step-by-step explanation:

V  W  

1 x 50     = 50

1.5 x 33 1/3 = 50

2 x 25     = 50

2.5 x 20     = 50

3 x 16 2/3 = 50

3.5 x 14 2/7 = 50

4 x 12 1/2 = 50

4.5 x 11 1/9 = 50

5 x 10     = 50

5.5 x 9     = 50

6 x 8 1/3 = 50

6.5 x 7 2/3 = 50

7 x 7 1/7 = 50

7.5 x 6 2/3 = 50

8 x 6 1/4 = 50

8.5 x 5 8/9 = 50

9 x 5 5/9 = 50

9.5 x 5 1/4 = 50

10 x 5     = 50

10.5 x 4 3/4 = 50

11 x 4 5/9 = 50

11.5 x 4 1/3 = 50

12 x 4 1/6 = 50

12.5 x 4     = 50

13 x 3 6/7 = 50

13.5 x 3 5/7 = 50

14 x 3 4/7 = 50

3 different problems.

solve b - 4 = 3.

solve -32 = 8w

solve n
— = -4
7

Answers

Answer:

see below

Step-by-step explanation:

b - 4 = 3

Add 4 to each side

b - 4+4 = 3+4

b = 7

-32 = 8w

Divide by 8

-32/8 = 8w/8

-4 = w

n/7 = -4

Multiply by 7 to each side

n/7 *7 = -4*7

n = -28

Answer:

1. b = 7

2. w = -4

3. n = -28

Step-by-step explanation:

1. b - 4 = 3

Tip: Whenever there’s a number being subtracted or added, it’s best to make that number a 0

Step 1. Add 4 to both sides

b - 4 = 3

b + 4 = +4

b = 7

—————————————
2. -32 = 8w

Step 1. Isolate the variable
(by dividing both sides by 8)

w = -32/8

w = -4

—————————————
3. n/7 = -4

Step 1. Multiply 7 x -4

n = -28

A chef mix his salt and pepper. If he put 2/3 cup of salt and 1/2 cup of pepper in his shaker, what is the ratio of salt to pepper ?

Answers

Answer:

4 to 3

Step-by-step explanation:

The ratio of salt to pepper is

2/3 to 1/2

Multiply both fractions by 6

6 * 2/3 to 6 * 1/2

4 to 3

The bar graph shows the median income for families in the United States from 1993 through 2000.
Which two consecutive years saw the largest increase in median income?

A. 1994–1995

B. 1997–1998

C. 1998–1999

D. 1999–2000

Answers

The two consecutive years in which there is the largest increase in the median income is option B i.e. 1997-1998.

Given that

In 1994-1995, the median income is 37,500 and 38,500.In 1997-1998, the median income is 39,700 and 41,000.In 1998-1999, the median income is 41,000 and 42,200.In 1999-2000, the median income is 42,500 and its less.

So after analysis, we can conclude that The two consecutive years in which there is the largest increase in the median income is option B i.e. 1997-1998.

Learn more about the bar graph here: brainly.com/question/14894834

help me please !! find the domain and range of the function represented by the graph. determine wether the domain is discrete or continuous.

Answers

Answer:

Domain is the whole R and the range is R+. The domain is continuous

Please help i do not understand this question!

Answers

Answer:

Distributive property

Step-by-step explanation:

The rule being shown here is the distributive property, because the first binomial is taken apart and separately multiplied with the other binomial.

if f(x) = 2x²+2x-1, g(x) = x²+5x+2 f(x)=g(x), find the value of x​

Answers

Answer:

Possible Answers: –5. –21. 7. –1

Answer:

just put the values and simpilyfy it untill you get the value of X

hence, the value of X is either 3 or, -1

the train regularly travels 135 miles in 60 minutes.

What is the speed, in miles per minute, on this portion of the route?

Answers

Answer:

2.25 miles per minute

Step-by-step explanation:

Speed = Distance/Time

135/60

2.25 miles per minute

Answer from Gauthmath

91. Jack can read 45 pages of his book in one and a
half hours. At that rate, how long will it take him to
read the entire 300-page book?

Answers

Answer:

10 hours

Step-by-step explanation:

45 pages per 1 hour and 30 minutes,  or 60 min +30 min =90 min

90 minutes / 45 pages has to be equal to an equivalent fraction where we have 300 pages

90 minutes /45 pages = ? minutes / 300 pages , multiply both sides by 300

? minutes = 90*300/45 = 600 minutes

to read 300-page book will take 600 minutes = 10 hours

if measure of three angles of a quadrilateral are 65 degree 95 degree and 45 degree then find the measure of the fourth​ angle?

Answers

Answer:

155 degrees

Step-by-step explanation:

Hi there!

The sum of the interior angles of a quadrilateral is always 360 degrees. To find the measure of the fourth angle, subtract 65, 95 and 45 from 360:

360-65-95-45

= 155

Therefore, the measure of the fourth angle is 155 degrees.

I hope this helps!

Answer:

155 degrees

Step-by-step explanation:

A quadrilateral has a total internal angle sum of 360 so we can set up an equation where x is the fourth angle

360=65+95+45+x

360=205+x

x=155

Look at this cube
If the side lengths are halved, then which of the following statements about its surface area will be true?

Answers

Answer:

the new surface Will be 1/4 pf the old surface

Step-by-step explanation:

because the surface with 7 in Is about 294 and with 3.5 (1/2 of 7) Is 73.5.

so 294:4 Is 73.5

excuse me for my bad English but I'm italian

will mark brainliest help!

In parallelogram ABCD, m∠B = 105°.

What is the value of x. (Round your answer to the nearest tenth, if necessary.)

Answers

Answer:

52.5

Step-by-step explanation:

Since ED ≈ AD, ∆AED is a isosceles triangle where <EAD and <AED has same angle measurement

Now, m<B = 105°, since ABCD is a parallelogram, <B = <D

so, m<ADC = 105° so, m<ADE = 180°-105° = 75°

Now, m<EAD+m<AED+m<ADE = 180°

or, m<AED+m<AED+m<ADE = 180°

or, x+x+75°=180°

or, 2x=180+75°

or, 2x=105°

or, x=52.5°

Someone please help me find y if you don’t mind Thankyouu so much

Answers

Answer:

Step-by-step explanation:

Decreasing rate = 12% = 0.12

y = 2400 * (1- 0.12)^x = 2400*(0.88)^x

x = 8 years

[tex]y = 2400 *(0.88)^{8}\\\\= 2400*0.36\\\\= 863[/tex]

Find the cross product (7,9, 6)x 44, 1, 5). Is the resulting vector perpendicular to the given vectors?
a. (-57, 43, 33); yes
c. (33. – 59, - 57); no
b. (39, – 59,43), yes
d. (33, - 57, 49); no

Answers

Answer:

Option B, (39, -59, 43)

Answered by GAUTHMATH

The difference of a number and 11

Answers

Answer:

x - 11

Step-by-step explanation:

"A number" is the same thing as a variable.

Answer:

x-11

Step-by-step explanation:

42 deliveries in 3 hours = deliveries per hour

Answers

Answer:

14 deliveries per hour

Step-by-step explanation:

42/3

14

The person makes 14 deliveries per hour

Answer:

14

Step-by-step explanation:

42 / 3 = 14

If 1 angle is four time of another angle in linear pair find the angles. Please do it fast as you can

Answers

Answer:

let the angle be x then other angle is 4x

so,

x + 4x = 180

5x = 180

x = 36

so other angle I.e 4x = 4 × 36 = 144

Let one angle be x

Other angle=4x

Both are linear pair hence their sum will be 180

[tex]\\ \sf \longmapsto x+4x=180[/tex]

[tex]\\ \sf \longmapsto 5x=180[/tex]

[tex]\\ \sf \longmapsto x=\dfrac{180}{5}[/tex]

[tex]\\ \sf \longmapsto x=30[/tex]

[tex]\\ \sf \longmapsto 5x=5(30)=150[/tex]


Four times an angle is equal to half of its supplement. Find the measures of both angles.

Answers

Answer:

The bigger angle is 144 degrees and the smaller one is 36 degrees

Find the place value of 8 in 2456.1387.
Tenths
Hundredths
Thousandths
Thousands

Answers

Answer: Third Choice. Thousandths

Step-by-step explanation:

Concept:

Here, we need to know the order and name of each place value.

Please refer to the attachment below for the specified names.

Solve:

STEP ONE: Orde and name each place

2 ⇒ One Thousands

4 ⇒ Hundreds

5 ⇒ Tens

6 ⇒ Ones

.

1 ⇒ Tenths

3 ⇒ Hundredths

8 ⇒ One Thousandths

7 ⇒ Ten Thousandths

STEP TWO: Find the number [8] in the number

As we can see from the list above, 8 is at the right of the decimal point, thus, the place value is Thousandths.

Hope this helps!! :)

Please let me know if you have any questions

find the radius of a circle for which an arc 6 cm long subtends an angle of 1/3 radians at the center?
plz some one can help to solve the question??​

Answers

Step-by-step explanation:

Eueydhhdgdgdbdbddbdbhd

Answer:

Hello,

[tex]R=\dfrac{18}{\pi}\ (cm)[/tex]

Step-by-step explanation:

[tex]Formula: \ L=\theta*R\\[/tex]

[tex]R=\dfrac{6}{\dfrac{\pi}{3} } =\dfrac{6*3}{\pi} =\dfrac{18}{\pi}\ (cm)[/tex]

help please help......​

Answers

Answer:

[tex]{ \sf{( \sec \theta - \csc \theta)(1 + \tan \theta + \cot \theta) }} \\ = { \sf{( \sec \theta + \tan \theta \sec \theta + \cot \theta \sec \theta) - ( \csc \theta - \csc \theta \tan \theta - \csc \theta \cot \theta)}} \\ = { \sf{( \sec \theta + \sec \theta \tan \theta + \csc \theta) - ( \csc \theta - \sec \theta - \csc \theta \cot \theta)}} \\ = { \sf{ \sec \theta \tan \theta + \csc \theta \cot \theta }} \\ { \bf{hence \: proved}}[/tex]

TO BE ANSERED ASAP
If n = 4, then 9ˆ8 ÷ 9 n is equal to__________________.

Answers

Answer:

1,195,742.25 (final answer since instructions did not include rounding up or anything)

Step-by-step explanation:

n = 4

The given expression is:

9^8 ÷ 9n

USE PEMDAS order of operations and solve:

9^8 ÷ 9n

= 43046721 ÷ 9n

Substitue n with 4 and solve:

43046721 ÷ 9(4)

= 43046721 ÷ 36

= 1,195,742.25

Answer:

9^4

Step-by-step explanation:

We know that a^b ÷ a^c = a^ (b-c)

9^8 ÷ 9^4

9^(8-4)

9^4

Other Questions
Sometimes people say silly things they do not intend to be mean.Identify the choice that corrects this run-on sentence. . Using the identity (a + b) = (a + 2ab + b), evaluate 112 Jesse travels 3.0 meters east and then turns and travels 4.0 meters north. The trip requires 35 seconds. What is his velocity? **25 POINTS**Which of the following lists of ordered pairs is a function? A skydiver slows down from 65 m/s to 5 m/s by opening the parachute. If thistakes 0.75 seconds, what is the skydiver's acceleration?A. 45 m/s2 upB. 80 m/s2 upC. 45 m/s2 downD. 80 m/s2 down Graph the line that passes through (5, 5), and is perpendicular to a line whose slope is 2. Express as index formlog 2 64 = 6 ANSWER AND ILL GIVE BRAINLIEST Which of the following statements can you infer would happen If one organ system stopped working?a. The other organ systems would have to work harder to make up for the broken system b. The other organ systems would stop working too.c. The other organ systems would still work on their own but might have to work harder to complete their function correctly. In the small country of Economerica, there are 6 thousand people employed, 1 thousand people unemployed, and 3 thousand people of working age not included in the labor force. One thousand students graduate from the university and seek jobs in the country, but not one graduate is able to find one. Half of the new graduates believe there is no chance of finding a job and return home to live with their parents. Ceteris Paribus, what would the new unemployment rate be in Economerica?A. 14.29%B. 20.00%C. 25.00%D. 45.45% What is the slope of a roof on a house that has a vertical height of 2.4 feet from the ceiling of the top floor to the top of the pitch and a length of 8.2 feet from the center of the edge of the house? 5. Describe three ways that music can be used in television. (6 points) ASAP HELP PLS NO WRONG ANSWERS------------ PLS HELP ME ON THIS QUESTION I WILL MARK YOU AS BRAINLIEST IF YOU KNOW THE ANSWER PLS GIVE ME A STEP BY STEP EXPLANATION!! For the function G defined by G(x) = 5x + 3, find G(2) A.Yes, since the slopes are the same and the y-intercepts are the same.B.No, since the y-intercepts are different.C.Yes, since the slopes are the same and the y-intercepts are different.D.No, since the slopes are different. 6) Frazer cycles the first 20 miles at an average speed of 21mph. The secondpart is more uphill, and he only manages 13mph. By what percentage did hisspeed decrease?How to solve This is the lastWhat are the vestigal organs present in human body?? Write each percent as a fraction in simple form 15 3/5% what is bacteria caused by Which of the following is true? A. A strong argumentative essay should contain secondary sources and not primary sources. B. A strong argumentative essay should contain primary sources and not secondary sources. C. A strong argumentative essay should contain primary and secondary sources. D. A strong argumentative essay should never contain primary and secondary sources.